ChaseDream

标题: 求助 og048 [打印本页]

作者: cissie0128    时间: 2006-2-21 23:59
标题: 求助 og048
48. A researcher discovered that people who have low levels of immune-system activity tend to score much lower on tests of mental health than do people with normal or high immune-system activity. The researcher concluded from this experiment that the immune system protects against mental illness as well as against physical disease


.


The researcher’s conclusion depends on which of the following assumptions?


A. High immune-system activity protects against mental illness better than normal immune-system activity does.


B. Mental illness is similar to physical disease in its effects on body systems.


C. People with high immune-system activity cannot develop mental illness.


D. Mental illness does not cause people’s immune-system activity to decrease. D


E. Psychological treatment of mental illness is not as effective as is medical treatment.



48.


The researcher concludes from the association of low immune-system activity with low mental-health sores that, in effect, immune system activity can inhibit mental illness. If, contrary to D, mental illness can depress immune-system activity, the association mentioned does not support the researcher’s conclusion. So D must be assumed.



这里是在取非么,对d选项取非后是心理疾病会引起免疫系统下降,和原文有什么矛盾的呢?


作者: dreamchaserdan    时间: 2006-2-22 23:10
标题: 回复:(cissie0128)求助 og048

呵呵,今天刚好看到这道题。我的理解如下:


这是个因果型结论,OG的解释认为low levels of immune-system activity和lower test score of mental health同时存在,所以认为结论这两者有因果关系,而且是前者引起后者。作为assumption题的答案D,应该是否认因果倒置的可能性,也就是说不是后者引起前者。如果将这个答案取非,的确是后者引起前者,题目中的结论就是错误的。


作者: cissie0128    时间: 2006-2-23 14:29

差不多了,谢谢哦






欢迎光临 ChaseDream (https://forum.chasedream.com/) Powered by Discuz! X3.3